3
$\begingroup$

Let a differentiable real function $f$ be such that $\left|f'(x)\right|\le|f(x)|$. Prove that $f(x)$ has the same sign for all $x$. (Let 0ドル$ and any real number have the same sign)

  1. If $f$ has no zeros then we are done because $f$ is continuous.

  2. If $f$ has zeros, WLOG let $f(0)=0$. One way is to consider $F(x)=\mathrm e^{-2x}f^2(x)$, where $$F'(x)=2\mathrm e^{-2x}\left(f(x)f'(x)-f^2(x)\right)\le0$$ due to $\left|f'(x)\right|\le|f(x)|$. The rest is simple.

But I did not think of such $F(x)$. I wanted to know whether my idea as follows can give a complete proof. (I don't want to see answers that continue the approach above!!!)

Let $f$ have zeros. If the zeros are dense, then by continuity $f\equiv0$. Otherwise, I think we can find $f(x_0)=0$ such that both $f$ and $f'$ are positive/negative on $(x_0,x_0+\delta)$ (I did not prove this!). WLOG positive, consider $g(x)=\mathrm e^{-x}f(x)$, then $g'(x)=\mathrm e^{-x}(f'(x)-f(x))\le0$ so $g(x)$ is a decreasing function which gives a contradiction.

asked Nov 19 at 10:44
$\endgroup$
11
  • 1
    $\begingroup$ Why does g(x) being decreasing give a contradiction? $\endgroup$ Commented Nov 19 at 10:51
  • $\begingroup$ Because $g$ and $f$ have the same sign. @MathIsAlwaysRight $\endgroup$ Commented Nov 19 at 11:01
  • $\begingroup$ sorry if I'm misunderstanding something, but what if g just decreases towards some asymptote? It doesn't necessarily have to change its sign, does it? $\endgroup$ Commented Nov 19 at 11:04
  • $\begingroup$ @MathIsAlwaysRight But $g(x_0)=0$. $\endgroup$ Commented Nov 19 at 11:06
  • $\begingroup$ Why $f'$ would be positive/negative on $(x_0,x_0+\delta)$? It is false in general (and useless for your conclusion). $\endgroup$ Commented Nov 19 at 11:41

1 Answer 1

3
$\begingroup$

Let $I=\{x:\ f(x)=0\}$. If $I=\emptyset$, we're done since $f$ is continuous. Because of continuity, $I$ consists of closed intervals, hence $\mathbb R\setminus I$ is open and thus is a (at most countable) collection of open intervals. Let $b$ belong to some such interval $(a,c)$ where $c\in(a,+\infty]$ and assume w.l.o.g. that $f(b)>0$; hence $f(x)>0$ for all $x\in(a,b]$, $f(a)=0$.

Now, $$ \int_a^b \frac{f'(x)}{f(x)}dx=\ln(f(b))-\ln(f(a))=+\infty $$ and at the same time $$ \int_a^b \frac{f'(x)}{f(x)}dx\le \int_a^b \frac{|f'(x)|}{f(x)}dx \le \int_a^b 1 dx=b-a $$ Contradiction.

answered Nov 19 at 11:16
$\endgroup$
4
  • $\begingroup$ Why is the second inequality? $\endgroup$ Commented Nov 19 at 11:27
  • 1
    $\begingroup$ There is a $\le$ missing in front of $\int_a^b 1 dx$. $\endgroup$ Commented Nov 19 at 11:31
  • 1
    $\begingroup$ Should be $\le$ and not $=,ドル right? $\endgroup$ Commented Nov 21 at 7:16
  • 1
    $\begingroup$ Thanks, @Gary ! $\endgroup$ Commented Nov 21 at 8:56

You must log in to answer this question.

Start asking to get answers

Find the answer to your question by asking.

Ask question

Explore related questions

See similar questions with these tags.